Difference between revisions of "2021 AMC 10A Problems/Problem 2"

(Problem)
m (Solution 1)
(46 intermediate revisions by 19 users not shown)
Line 1: Line 1:
 
==Problem==
 
==Problem==
Lorem ipsum dolor sit amet, consectetur adipiscing elit, sed do eiusmod tempor incididunt ut labore et dolore magna aliqua. Nam libero justo laoreet sit amet. Eget arcu dictum varius duis at consectetur. Amet venenatis urna cursus eget nunc. Feugiat nisl pretium fusce id velit. Quam elementum pulvinar etiam non quam lacus. Feugiat nisl pretium fusce id velit. Felis eget nunc lobortis mattis aliquam faucibus purus. Iaculis urna id volutpat lacus laoreet non curabitur gravida arcu. Et ultrices neque ornare aenean euismod elementum nisi quis eleifend. Tellus id interdum velit laoreet id donec ultrices. Nascetur ridiculus mus mauris vitae ultricies. Ut placerat orci nulla pellentesque dignissim enim sit. Varius vel pharetra vel turpis. Aliquam id diam maecenas ultricies mi eget mauris. At tempor commodo ullamcorper a lacus vestibulum sed arcu non. Nulla malesuada pellentesque elit eget gravida cum sociis. Auctor augue mauris augue neque. Risus ultricies tristique nulla aliquet enim.
+
Portia's high school has <math>3</math> times as many students as Lara's high school. The two high schools have a total of <math>2600</math> students. How many students does Portia's high school have?
  
==Solution==
+
<math>\textbf{(A)} ~600 \qquad\textbf{(B)} ~650 \qquad\textbf{(C)} ~1950 \qquad\textbf{(D)} ~2000\qquad\textbf{(E)} ~2050</math>
==See also==
+
 
 +
==Solution 1==
 +
The following system of equations can be formed with <math>p</math> representing the number of students in Portia's high school and <math>l</math> representing the number of students in Lara's high school.
 +
<cmath>p=3l</cmath>
 +
<cmath>p+l=2600</cmath>
 +
Substituting <math>p</math> with <math>3l</math> we get <math>4l=2600</math>. Solving for <math>l</math>, we get <math>l=650</math>. Since we need to find <math>p</math> we multiply <math>650</math> by 3 to get <math>p=1950</math>, which is <math>\boxed{\text{C}}</math>
 +
 
 +
-happykeeper
 +
 
 +
==Solution 2 (One Variable)==
 +
Suppose Lara's high school has <math>x</math> students. It follows that Portia's high school has <math>3x</math> students. We know that <math>x+3x=2600,</math> or <math>4x=2600.</math> Our answer is <cmath>3x=2600\left(\frac 34\right)=650(3)=\boxed{\textbf{(C)} ~1950}.</cmath>
 +
 
 +
~MRENTHUSIASM
 +
 
 +
==Solution 3 (Arithmetics)==
 +
Clearly, <math>2600</math> students is <math>4</math> times as many students as Lara's high school. Therefore, Lara's high school has <math>2600\div4=650</math> students, and Portia's high school has <math>650\cdot3=\boxed{\textbf{(C)} ~1950}</math> students.
 +
 
 +
~MRENTHUSIASM
 +
 
 +
==Solution 4 (Answer Choices)==
 +
===Solution 4.1 (Quick Inspection)===
 +
The number of students in Portia's high school must be a multiple of <math>3.</math> This eliminates <math>\textbf{(B)},\textbf{(D)},</math> and <math>\textbf{(E)}</math>. Since <math>\textbf{(A)}</math> is too small (as <math>600+600/3<2600</math> is clearly true), we are left with <math>\boxed{\textbf{(C)} ~1950}.</math>
 +
 
 +
~MRENTHUSIASM
 +
 
 +
===Solution 4.2 (Plug in the Answer Choices)===
 +
For <math>\textbf{(A)},</math> we have <math>600+\frac{600}{3}=800\neq2600.</math> So, <math>\textbf{(A)}</math> is incorrect.
 +
 
 +
For <math>\textbf{(B)},</math> we have <math>650+\frac{650}{3}=866\frac{2}{3}\neq2600.</math> So, <math>\textbf{(B)}</math> is incorrect.
 +
 
 +
For <math>\textbf{(C)},</math> we have <math>1950+\frac{1950}{3}=2600.</math> So, <math>\boxed{\textbf{(C)} ~1950}</math> is correct. For completeness, we will check choices <math>\textbf{(D)}</math> and <math>\textbf{(E)}.</math>
 +
 
 +
For <math>\textbf{(D)},</math> we have <math>2000+\frac{2000}{3}=2666\frac{2}{3}\neq2600.</math> So, <math>\textbf{(D)}</math> is incorrect.
 +
 
 +
For <math>\textbf{(E)},</math> we have <math>2050+\frac{2050}{3}=2733\frac{1}{3}\neq2600.</math> So, <math>\textbf{(E)}</math> is incorrect.
 +
 
 +
~MRENTHUSIASM
 +
 
 +
==Video Solution #1(Setting Variables) ==
 +
 
 +
https://youtu.be/qNf6SiIpIsk?t=119
 +
~ThePuzzlr
 +
 
 +
==Video Solution #2(Solving by equation) ==
 +
 
 +
https://www.youtube.com/watch?v=aOpgeMfvUpE&list=PLexHyfQ8DMuKqltG3cHT7Di4jhVl6L4YJ&index=1
 +
~North America Math Contest Go Go Go
 +
 
 +
==Video Solution==
 +
https://youtu.be/xXx0iP1tn8k
 +
 
 +
- pi_is_3.14
 +
 
 +
==Video Solution (Simple)==
 +
https://youtu.be/DOtysU-a1B4
 +
 
 +
~ Education, the Study of Everything
 +
 
 +
==Video Solution 5==
 +
https://youtu.be/GwwDQYqptlQ
 +
 
 +
~savannahsolver
 +
 
 +
==Video Solution 6==
 +
https://youtu.be/50CThrk3RcM?t=66
 +
 
 +
~IceMatrix
 +
 
 +
==Video Solution (Problems 1-3)==
 +
https://youtu.be/CupJpUzKPB0
 +
 
 +
~MathWithPi
 +
 
 +
==See Also==
 
{{AMC10 box|year=2021|ab=A|num-b=1|num-a=3}}
 
{{AMC10 box|year=2021|ab=A|num-b=1|num-a=3}}
 
{{MAA Notice}}
 
{{MAA Notice}}

Revision as of 00:07, 22 February 2021

Problem

Portia's high school has $3$ times as many students as Lara's high school. The two high schools have a total of $2600$ students. How many students does Portia's high school have?

$\textbf{(A)} ~600 \qquad\textbf{(B)} ~650 \qquad\textbf{(C)} ~1950 \qquad\textbf{(D)} ~2000\qquad\textbf{(E)} ~2050$

Solution 1

The following system of equations can be formed with $p$ representing the number of students in Portia's high school and $l$ representing the number of students in Lara's high school. \[p=3l\] \[p+l=2600\] Substituting $p$ with $3l$ we get $4l=2600$. Solving for $l$, we get $l=650$. Since we need to find $p$ we multiply $650$ by 3 to get $p=1950$, which is $\boxed{\text{C}}$

-happykeeper

Solution 2 (One Variable)

Suppose Lara's high school has $x$ students. It follows that Portia's high school has $3x$ students. We know that $x+3x=2600,$ or $4x=2600.$ Our answer is \[3x=2600\left(\frac 34\right)=650(3)=\boxed{\textbf{(C)} ~1950}.\]

~MRENTHUSIASM

Solution 3 (Arithmetics)

Clearly, $2600$ students is $4$ times as many students as Lara's high school. Therefore, Lara's high school has $2600\div4=650$ students, and Portia's high school has $650\cdot3=\boxed{\textbf{(C)} ~1950}$ students.

~MRENTHUSIASM

Solution 4 (Answer Choices)

Solution 4.1 (Quick Inspection)

The number of students in Portia's high school must be a multiple of $3.$ This eliminates $\textbf{(B)},\textbf{(D)},$ and $\textbf{(E)}$. Since $\textbf{(A)}$ is too small (as $600+600/3<2600$ is clearly true), we are left with $\boxed{\textbf{(C)} ~1950}.$

~MRENTHUSIASM

Solution 4.2 (Plug in the Answer Choices)

For $\textbf{(A)},$ we have $600+\frac{600}{3}=800\neq2600.$ So, $\textbf{(A)}$ is incorrect.

For $\textbf{(B)},$ we have $650+\frac{650}{3}=866\frac{2}{3}\neq2600.$ So, $\textbf{(B)}$ is incorrect.

For $\textbf{(C)},$ we have $1950+\frac{1950}{3}=2600.$ So, $\boxed{\textbf{(C)} ~1950}$ is correct. For completeness, we will check choices $\textbf{(D)}$ and $\textbf{(E)}.$

For $\textbf{(D)},$ we have $2000+\frac{2000}{3}=2666\frac{2}{3}\neq2600.$ So, $\textbf{(D)}$ is incorrect.

For $\textbf{(E)},$ we have $2050+\frac{2050}{3}=2733\frac{1}{3}\neq2600.$ So, $\textbf{(E)}$ is incorrect.

~MRENTHUSIASM

Video Solution #1(Setting Variables)

https://youtu.be/qNf6SiIpIsk?t=119 ~ThePuzzlr

Video Solution #2(Solving by equation)

https://www.youtube.com/watch?v=aOpgeMfvUpE&list=PLexHyfQ8DMuKqltG3cHT7Di4jhVl6L4YJ&index=1 ~North America Math Contest Go Go Go

Video Solution

https://youtu.be/xXx0iP1tn8k

- pi_is_3.14

Video Solution (Simple)

https://youtu.be/DOtysU-a1B4

~ Education, the Study of Everything

Video Solution 5

https://youtu.be/GwwDQYqptlQ

~savannahsolver

Video Solution 6

https://youtu.be/50CThrk3RcM?t=66

~IceMatrix

Video Solution (Problems 1-3)

https://youtu.be/CupJpUzKPB0

~MathWithPi

See Also

2021 AMC 10A (ProblemsAnswer KeyResources)
Preceded by
Problem 1
Followed by
Problem 3
1 2 3 4 5 6 7 8 9 10 11 12 13 14 15 16 17 18 19 20 21 22 23 24 25
All AMC 10 Problems and Solutions

The problems on this page are copyrighted by the Mathematical Association of America's American Mathematics Competitions. AMC logo.png